Sabrina is 4 feet tall and casts a shadow that is 3. 5 feet tall. A nearby pole is 10 feet tall. How tall will the pole’s shadow be if Sabina and the pole are proportional? Leave your answer as a fraction

Answers

Answer 1

If Sabina and the pole are proportional, then the pole’s shadow will be 35/4 feet tall.

Since Sabrina is 4 feet tall and casts a 3.5-foot shadow, we can set up a proportion comparing the heights and shadow lengths: Sabrina's height (4 feet) / her shadow (3.5 feet) = pole's height (10 feet) / pole's shadow (x).

This proportion can be represented as: 4/3.5 = 10/x. To solve for x (the length of the pole's shadow), we can cross-multiply:

4 * x = 3.5 * 10

4x = 35

Now, divide both sides by 4:

x = 35/4

So, the pole's shadow will be 35/4 feet long when both Sabrina and the pole are proportional. This fraction represents the pole's shadow length in relation to the given heights and shadow lengths.

Learn more about proportion here: https://brainly.com/question/31188065

#SPJ11


Related Questions

PLEASE HELP Solve for f(x)!!

Answers

Answer:

8.81

Step-by-step explanation:

Substitute x for 7 and then solve normally

{2(7)^2+7-8}/(7)+4

{(2x49)+7-8}/11

98+7-8/11

97/11

8.81

Alejandro will deposit $1,750 in an account that earns 6. 5% simple interest every year. His sister Anallency will deposit $1,675 in an account that earns 7. 5% interest compounded annually. The deposits will be made on the same day, and no additional money will be deposited or withdrawn from the accounts. How much more money will Anallency have?

Answers

Anallency will have $134.46 more money than Alejandro after one year of their respective deposits.

How much more money will Anallency have than Alejandro after one year of their respective deposits?

Alejandro and Anallency are depositing money in separate accounts with different interest rates. Alejandro is depositing $1,750 in an account that earns a simple interest rate of 6.5% per year, while Anallency is depositing $1,675 in an account that earns a compounded interest rate of 7.5% per year. After one year of their respective deposits, Anallency will have $134.46 more money than Alejandro.

The difference in interest earned between the two accounts is the reason for the difference in money earned by Alejandro and Anallency. Alejandro's account earns a simple interest rate, which means that he earns 6.5% of his initial deposit every year, regardless of how much interest he has already earned.

On the other hand, Anallency's account earns a compounded interest rate, which means that she earns interest on both her initial deposit and on any interest earned in previous years.

The formula for calculating simple interest is I = Prt, where I is the interest earned, P is the principal (or initial deposit), r is the interest rate, and t is the time in years. Using this formula, we can calculate that Alejandro will earn $113.75 in interest after one year.

The formula for calculating compounded interest is A = P(1 + r/n)^(nt), where A is the amount of money at the end of the time period, P is the principal, r is the interest rate, n is the number of times the interest is compounded per year, and t is the time in years.

Using this formula, we can calculate that Anallency will have $248.21 more in her account than Alejandro after one year.

Learn more about Interest rate

brainly.com/question/13324776

#SPJ11

Selika give her garden a makeover. She spends money on plant,materials, and labour in the ratio of 1:5:12. She spends £848. 75. How much money does she spend on labour costs

Answers

Selika spends £565.85 on labour costs.

Given, Selika spends money on plants, materials, and labor in the ratio of 1:5:12 and spends a total of £848.75. We have to find how much money she spends on labor costs.

Let the amount of money Selika spends on plants be x. Then, the amount of money she spends on materials is 5x, and the amount of money she spends on labor is 12x.

The total amount of money she spends is £848. 75

x + 5x + 12x = 848.75

18x = 848.75

x = 848.75/18

x = 47.15

She spend on labour 12x = 12 × 47.15

= 565.85

Therefore, Selika spends £565.85 on labour costs.

Learn more about ratio here

brainly.com/question/13419413

#SPJ4

How many different simple random samples of size 4 can be obtained from a population whose size is 50?

Answers

The number of random samples, obtained using the formula for combination are 230,300 random samples

What is a random sample?

A random sample is a subset of the population, such that each member of the subset have the same chance of being selected.

The formula for combinations indicates that we get;

nCr = n!/(r!*(n - r)!), where;

n = The size of the population

r = The sample size

The number of different simple random samples of size 4 that can be obtained from a population of size 50 therefore can be obtained using the above equation by plugging in r = 4, and n = 50, therefore, we get;

nCr = 50!/(4!*(50 - 4)!) = 230300

The number of different ways and therefore, the number of random samples of size 4 that can be selected from a population of 50 therefore is 230,300 random samples.

Learn more on combination here: https://brainly.com/question/25718474

#SPJ1

Rotation of 180°, followed by a dilation with scale factor 5, followed by a reflection over the line y = x.
a. A' (15, -10) b.
A' (-15, 10)
C. A' (-10, 15)
d. A' (10, -15)

Answers

Answer:

A

Step-by-step explanation:

Let's call the length of each of the other two sides x. Since the triangle is isosceles, it has two sides of equal length. Therefore, the perimeter of the triangle can be expressed as 6 + x + x Simplifying this equation, we get  2x + 6 We know that the perimeter is 22 cm so we can set up an equation and solve for x. 22 = 2x + 6 Subtracting 6 from both sides, we get  16 = 2x Dividing both sides by 2, we get x=8

The bike sarah wants to buy is now 40% off. the original price is $150. decide if you are missing the percent, part or whole. then use the appropriate formula to find the discount amount

Answers

The discount amount of the bike Sarah wants to buy is $60. The calculation was done by using the formula: Discount = Original price x Percent off.

To find the discount amount of the bike, we need to use the formula

Discount = Original Price x Discount Rate

where Discount Rate = Percent Off / 100

We know that the original price of the bike is $150 and it is now 40% off. So, the discount rate is

Discount Rate = 40 / 100 = 0.4

Substituting these values in the formula, we get:

Discount = $150 x 0.4 = $60

Therefore, the discount amount of the bike is $60.

To know more about discount amount:

https://brainly.com/question/29181625

#SPJ4

one serving of almonds is 1/3 cup. booker bought 2 and 2/3 cups of almonds how many servings of almonds did booker buy

Answers

Booker bought 8 servings of almonds.

How many servings of almonds did Booker buy if he purchased 2 and 2/3 cups of almonds, and one serving of almonds is 1/3 cup?

One serving of almonds is equal to 1/3 cup.

To find how many servings of almonds Booker bought, we can divide the total amount of almonds he purchased by the amount in one serving:

2 and 2/3 cups of almonds = 8/3 cups of almonds

Number of servings = (total amount of almonds purchased) / (amount in one serving)

Number of servings = (8/3) / (1/3)

Number of servings = 8/3 x 3/1

Number of servings = 8

Learn more about servings

brainly.com/question/31639498

#SPJ11

Earthworm Rivals are building the set for

their new music video. There is a tower made

of 9 glowing bricks that stands 5. 4 meters tall. If each of the bricks is the same exact size,

how tall is each brick?

Answers

Since each of the bricks is the same exact size, then each brick is 0.6 meters tall.

To determine the height of each glowing brick, we need to divide the total height of the tower (5.4 meters) by the number of bricks (9). This gives us the average height of each brick.

Using the formula for division, we can write this as:

Height of each brick = Total height of tower / Number of bricks

Plugging in the given values, we get:

Height of each brick = 5.4 meters / 9 bricks

Simplifying this expression, we can cancel out the units of "bricks" to get:

Height of each brick = 0.6 meters

Therefore, each glowing brick in the tower is 0.6 meters tall.

Learn more about division here: https://brainly.com/question/30126004

#SPJ11

Lourdes could choose to pay an ATM fee to get cash or use a credit card to pay for groceries


that cost $48. The credit card charges interest if the balance is not paid at the end of the month.


Should she use the debit card or credit card? Justify your choice.

Answers

Keep in mind that if Lourdes is able to pay off her credit card balance at the end of the month, she won't be charged any interest, making the credit card the better option in that case.

To determine whether Lourdes should use a debit card or credit card, we need to consider the ATM fee and the potential interest charges from the credit card.

Step 1: Determine the ATM fee
Find out how much Lourdes would be charged for using the ATM to withdraw cash.

Step 2: Determine the interest rate on the credit card
Check the credit card's terms and conditions to find the annual percentage rate (APR). This will help us calculate the potential interest charges.

Step 3: Calculate the potential interest charges
Assuming Lourdes doesn't pay off the credit card balance by the end of the month, divide the APR by 12 to get the monthly interest rate. Multiply this rate by the $48 grocery cost to find the potential interest charges.

Step 4: Compare costs
Compare the ATM fee and potential interest charges to determine the cheaper option. If the ATM fee is less than the potential interest charges, Lourdes should use her debit card. If the potential interest charges are less than the ATM fee, she should use her credit card.

Keep in mind that if Lourdes is able to pay off her credit card balance at the end of the month, she won't be charged any interest, making the credit card the better option in that case.

Learn more about credit card,

https://brainly.com/question/26857829

#SPJ11

Tom Jones, a mechanic at Golden Muffler Shop, is able to install new mufflers at an average rate of 3 per hour (exponential distribution). Customers seeking this service, arrive at the rate of 2 per hour (Poisson distribution). They are served first-in, first-out basis and come from a large (infinite population). Tom only has one service bay.



a. Find the probability that there are no cars in the system.


b. Find the average number of cars in the system.


c. Find the average time spent in the system.


d. Find the probability that there are exactly two cars in the system

Answers

a. To find the probability that there are no cars in the system, we need to use the formula for the steady-state probability distribution of the M/M/1 queue:
P(0) = (1 - λ/μ)
where λ is the arrival rate (2 per hour) and μ is the service rate (3 per hour).
P(0) = (1 - 2/3) = 1/3 or 0.3333
Therefore, the probability that there are no cars in the system is 0.3333.

b. To find the average number of cars in the system, we can use Little's Law:
L = λW
where L is the average number of cars in the system, λ is the arrival rate (2 per hour), and W is the average time spent in the system.
We can solve for W by using the formula:
W = 1/(μ - λ)
W = 1/(3 - 2) = 1 hour
Therefore, the average number of cars in the system is:
L = λW = 2 x 1 = 2 cars

c. To find the average time spent in the system, we already calculated W in part b:
W = 1 hour

d. To find the probability that there are exactly two cars in the system, we need to use the formula for the steady-state probability distribution:
P(n) = P(0) * (λ/μ)^n / n!
where n is the number of cars in the system.
P(2) = P(0) * (λ/μ)^2 / 2!
P(2) = 0.3333 * (2/3)^2 / 2
P(2) = 0.1111 or 11.11%
Therefore, the probability that there are exactly two cars in the system is 11.11%.

To know more about Poisson's Distribution:

https://brainly.com/question/9123296

#SPJ11

25. a state study on labor reported that one-third of full-time teachers in the state also worked part time at another job. for those teachers, the average number of hours worked per week at the part-time job was 13. after an increase in state teacher salaries, a random sample of 400 teachers who worked part time at another job was selected. the average number of hours worked per week at the part-time job for the teachers in the sample was 12. 5 with standard deviation 6. 5 hours. is there convincing statistical evidence at the level of 0. 05, that the average number of hours worked per week at part-time jobs decreased after the salary increase? (a) no. the p-value of the appropriate test is greater than 0. 5. (b) no. the p-value of the appropriate test is less than 0. 5. (c) yes. the p-value of the appropriate test is greater than 0. 5. (d) yes. the p-value of the appropriate test is less than 0. 5. (e) not enough information is given to determine whether there is convincing statistical evidence

Answers

From the solution to the question that we have here, the answer is A. There is no solid proof that the number of hours worked dropped after the income rise.

Let μ be the population mean number of hours worked per week by teachers who work part-time jobs, after the salary increase. Here are the alternate and null hypotheses:

H0: μ = 13

H1: μ < 13

n = 400

[tex]\bar{X}=12.5[/tex]

μ = 13

Formula for the t-test statistics is

[tex]t = \frac{\bar{X}-\mu}{s/\sqrt{n} }[/tex]

t = (12.5 - 13)/(6.5/√400)

t = (- 0.5)/(6.5/20)

t = - 10/6.5

= -1.5388

Degree of freedom is 400 -1 = 399

α = 0.05

The p-value is p(t < -1.5385) = 0.062

The p-value exceeds the level of significance. Therefore, we unable to reject the null hypothesis.

Hence, option a is correct.

Learn more on statistics here:

brainly.com/question/19243813

#SPJ4

Please help!!! you are painting the roof of a shed that is 35 ft from the ground. you are going to place the base of a
ladder 12 ft from the shed. how long does the ladder need to be to reach the roof of the shed? use pencil and
paper. explain how shortening the distance between the ladder and the shed affects the height of the ladder. the ladder needs to be ____ ft long to reach the roof of the shed.

Answers

To find the length of the ladder needed to reach the roof of the shed that is 35 ft from the ground with the base of the ladder 12 ft from the shed, you can use the Pythagorean theorem. The theorem states that in a right-angled triangle, the square of the length of the hypotenuse (the ladder, in this case) is equal to the sum of the squares of the other two sides (the height and the distance from the shed).

Step 1: Identify the sides of the triangle.
- Height (a): 35 ft (vertical side)
- Distance from the shed (b): 12 ft (horizontal side)
- Ladder length (c): Hypotenuse

Step 2: Apply the Pythagorean theorem.
- a² + b² = c²
- 35² + 12² = c²

Step 3: Calculate the squares and sum them.
- (35 * 35) + (12 * 12) = c²
- 1225 + 144 = c²
- 1369 = c²

Step 4: Find the length of the ladder (c).
- c = √1369
- c = 37

The ladder needs to be 37 ft long to reach the roof of the shed.

Shortening the distance between the ladder and the shed will affect the height of the ladder by making it steeper. This will cause the ladder to be higher above the ground, but it may also make it less stable and more difficult to climb.

To know more about Pythagorean theorem refer here

https://brainly.in/question/47213677#

#SPJ11

Find the length of the midsegment of the trapezoid

2n-2
N+12
4n+6

Answers

Answer: To find the length of the midsegment of a trapezoid, you need to add the lengths of the two bases together and divide by 2.

The length of the top base is 2n-2 and the length of the bottom base is 4n+6.

Adding the two bases together gives:

(2n-2) + (4n+6) = 6n + 4

Dividing by 2 gives:

(6n + 4) / 2 = 3n + 2

Therefore, the length of the midsegment of the trapezoid is 3n + 2.

Draw the image of ABC under the translation by 1 unit to the right and 4 units up

Answers

The image of ABC under the translation by 1 unit to the right and 4 units up is the triangle with vertices A', B', and C'.

What is Triangle?

A triangle is a closed two-dimensional geometric shape with three straight sides and three angles. It is one of the basic shapes in geometry and has been studied since ancient times. To translate a figure by a given vector, you need to move each point of the figure by the same amount and in the same direction as the vector. In this case, we want to translate ABC by a vector of (1, 4), which means we need to move each point of ABC one unit to the right and four units up.

Let's say the coordinates of the points of ABC are:

A = [tex]\rm (x_1, y_1)[/tex]

B = [tex]\rm (x_2, y_2)[/tex]

C = [tex]\rm (x_3, y_3)[/tex]

To translate ABC by the vector (1, 4), we add 1 to each x-coordinate and 4 to each y-coordinate:

A' = [tex]\rm (x_1 + 1, y_1 + 4)[/tex]   (x₁ + 1, y₁ + 4)

B' = [tex]\rm (x_2 + 1, y_2 + 4)[/tex]   (x₂ + 1, y₂ + 4)

[tex]\rm C' = (x_3 + 1, y_3 + 4)[/tex]  (x₃ + 1, y₃ + 4)

Therefore, The image of ABC under the translation by 1 unit to the right and 4 units up is the triangle with vertices A', B', and C'.

To learn more about Triangle from given link.

https://brainly.com/question/30599944

#SPJ1

Brainliest if correct!_A particle is projected vertically upwards from a fixed point O. The speed of projection is u m/s. The particle returns to O 4 seconds later. Find:

a) the value of u

b) the greatest height reached by the particle

c) the total time of which the particle is at a height greater than half its greatest height

Thank you so much!

Answers

The value of the velocity, u is 19.6 m/s.

The greatest height reached by the particle is 19.6 m.

The total time during which the particle is at a height greater than half its greatest height is 2.33 s.

What is the value of the velocity, u?

a) To find the value of the velocity, u, we can use the formula for the time of flight of a vertically projected particle:

t = 2u/g

Since the particle returns to the same point after 4 seconds, we have:

2t = 4

Substituting the value of t in the first equation, we get:

u = gt/2 = 9.8 x 2

u = 19.6 m/s

b) To find the greatest height reached by the particle, we can use the formula for the maximum height reached by a vertically projected particle:

h = u^2/2g

Substituting the value of u, we get:

h = 19.6^2/(2 x 9.8)

h = 19.6 m

c) To find the total time during which the particle is at a height greater than half its greatest height, we can first find the height at which the particle is at half its greatest height:

h/2 = (u^2/2g)/2 = u^2/4g

Substituting the value of u, we get:

h/2 = 19.6^2/(4 x 9.8) = 24.01 m

So, the particle is at a height greater than half its greatest height when it is above 24.01 m.

Next, we can find the time taken by the particle to reach this height:

h = ut - (1/2)gt^2

24.01 = 19.6t - (1/2)9.8t^2

Solving this quadratic equation, we get:

t = 2.33 s or t = 4.10 s

The particle takes 2.33 s to reach a height of 24.01 m, and it takes another 1.67 s (4 - 2.33) to return to the ground.

Learn more about velocity at: https://brainly.com/question/80295?source=archive

#SPJ1

Find the critical points for the function f(x, y) = x³ + y³ – 9x² – 3y - 6 = and classify each as a local maximum, local minimum, saddle point, or none of these. critical points: (give your points as a comma separated list of (x,y) coordinates.) classifications: (give your answers in a comma separated list, specifying maximum, minimum, saddle point, or none for each, in the same order as you entered your critical points)

Answers

The critical points and their classifications are:

(0, 1) - saddle point

(0, -1) - saddle point

(6, 1) - local minimum

(6, -1) - local minimum

To find the critical points of the function f(x, y) = x³ + y³ – 9x² – 3y - 6, we need to find the points where the partial derivatives of f with respect to x and y are zero.

∂f/∂x = 3x² - 18x = 3x(x - 6)

∂f/∂y = 3y² - 3 = 3(y² - 1)

Setting these partial derivatives equal to zero and solving for x and y, we get:

x = 0 or x = 6

y = ±1

So the critical points are (0, 1), (0, -1), (6, 1), and (6, -1).

To classify each critical point, we need to compute the second partial derivatives of f:

∂²f/∂x² = 6x - 18

∂²f/∂y² = 6y

∂²f/∂x∂y = 0

At (0, 1):

∂²f/∂x² = -18 < 0 (concave down)

∂²f/∂y² = 6 > 0 (concave up)

So (0, 1) is a saddle point.

At (0, -1):

∂²f/∂x² = -18 < 0 (concave down)

∂²f/∂y² = 6 > 0 (concave up)

So (0, -1) is a saddle point.

At (6, 1):

∂²f/∂x² = 18 > 0 (concave up)

∂²f/∂y² = 6 > 0 (concave up)

So (6, 1) is a local minimum.

At (6, -1):

∂²f/∂x² = 18 > 0 (concave up)

∂²f/∂y² = 6 > 0 (concave up)

So (6, -1) is a local minimum.

For more such questions on Critical points.

https://brainly.com/question/30889798#

#SPJ11

Terrell arranges x roses at $3. 50 each with 10 carnations at $2. 25 each. He makes a bouquet of flowers that averages $3. 00 per flower. Choose an equation to model the situation

Answers

The equation models the situation described in the problem is 3.50x + 2.25(10) = 3 (x + 10) . The correct answer is C.

To model the situation described in the problem, we need to use an equation that represents the total cost of the flowers in terms of the number of roses (x) and the number of carnations (10). Let's assume that the cost of each flower is proportional to its price, and that the average cost per flower is the total cost divided by the total number of flowers (x + 10).

The cost of x roses at $3.50 each is 3.50x, and the cost of 10 carnations at $2.25 each is 2.25(10) = 22.50. Therefore, the total cost of the bouquet is:

Total cost = 3.50x + 22.50

The average cost per flower is given by:

Average cost = Total cost / (x + 10)

We are told that the average cost per flower is $3.00, so we can set up an equation:

3.00 = (3.50x + 22.50) / (x + 10) or  3.50x + 2.25(10) = 3 (x + 10)

This equation models the situation described in the problem. We can solve for x to find the number of roses needed to make a bouquet that meets the given conditions. The correct answer is C.

Your question is incomplete but most probably your full question attached below

To learn more about equation visit:

brainly.com/question/21835898

#SPJ11

10 foot ladder is leaning against a vertical wall when Jack begins
pulling the foot of the ladder away from the wall at a rate of 0.5
fr/s. how fast is the top of the ladder sliding down the wall?

Answers

We can use the Pythagorean theorem to relate the distances between the ladder, wall, and ground. Let's call the distance from the foot of the ladder to the wall "x", and the distance from the top of the ladder to the ground "y". Then, we know that:

x^2 + y^2 = 10^2

We can differentiate this equation with respect to time to get:

2x(dx/dt) + 2y(dy/dt) = 0

We're interested in finding dy/dt, the rate at which the top of the ladder is sliding down the wall. We know that dx/dt = 0.5 ft/s, so we can plug in these values and solve for dy/dt:

2x(dx/dt) + 2y(dy/dt) = 0
2(8)(0.5) + 2y(dy/dt) = 0 (since x = 8 based on the Pythagorean theorem)
dy/dt = -4 ft/s

So the top of the ladder is sliding down the wall at a rate of 4 ft/s.
When the 10-foot ladder is leaning against a vertical wall, it forms a right-angled triangle with the wall and the ground. As Jack pulls the foot of the ladder away from the wall at a rate of 0.5 ft/s, the top of the ladder slides down the wall. To find the rate at which the top of the ladder slides down, we can use the Pythagorean theorem:

a^2 + b^2 = c^2

where a is the distance from the foot of the ladder to the wall, b is the height of the ladder's top from the ground, and c is the length of the ladder (10 feet).

Differentiating both sides with respect to time (t), we get:

2a(da/dt) + 2b(db/dt) = 0

We know that da/dt = 0.5 ft/s. We need to find db/dt, which is the rate at which the top of the ladder slides down the wall. To do this, we need to find the values of a and b at a given moment. Since the problem doesn't provide this information, it's not possible to determine the exact value of db/dt. However, if you have the values of a and b, you can plug them into the equation and solve for db/dt.

learn more about Pythagorean theorem here: brainly.com/question/343682

#SPJ11

Here is an inequality: -2x > 10.
1. List some values for x that would make this inequality true.

2. How are the solutions to the inequality -2x [tex]\geq[/tex] 10 different fomt the solutions to -2x > 10? Explain your reasoning.

Answers

Therefore , the solution of the given problem of inequality comes out to be the solutions x = -6 or x = -10 would be acceptable.

What exactly is an inequality?

Algebra, which lacking a symbol for this difference, can represent it using a pair or group of numbers. Equity usually comes after equilibrium. Inequality is bred by the persistent gap of standards. Equality and disparity are not the same thing. As was my least preferred symbol, notwithstanding knowing that the pieces are often not connected or close to one another. (). No matter how small the variations, they all affect value.

Here,

Finding values of x that cause the left side of the inequality to be bigger than the right side is necessary to make the inequality -2x > 10 true. Divide both sides by -2 and invert the inequality sign to achieve this:

=> -2x > 10

=> x < -5

The inequality is therefore true for any value of x that is less than -5. For instance, the solutions x = -6 or x = -10 would be acceptable.

To know more about inequality visit:

https://brainly.com/question/29914203

#SPJ1

Be Precise The base of a triangle is 2 ft. The
height of the triangle is 15 in. What is the area
of the triangle in square inches?

Answers

Thus, the area of triangle for the given values of height and base is found as:  180 sq. in.

Explain about the conversion units:

A number of steps are involved in the Unit of Conversion process, which involves multiplying or dividing by a numerical factor. There are numerous ways to measure things like weight, separation, and temperature.

Unit conversion is the process of changing the unit of measurement for a comparable quantity by multiplying or dividing by conversion factors.

Scientific notation is used to express the units, which are then translated into numerical values in accordance with the amounts.

Given data:

base of triangle b = 2 ft.Height h = 15 in.

We know that,

1 foot  = 12 in.

2 feet = 12*2 = 24 in.

Area of triangle = 1/2 * b * h

Area of triangle = 1/2 * 24 * 15

Area of triangle = 12* 15

Area of triangle = 180 sq. in

Thus, the area of triangle for the given values of height and base is found as:  180 sq. in.

know more about the conversion units:

https://brainly.com/question/13016491

#SPJ1

A professional athlete wants to tile his bedroom in solid gold. Each square tile will be 16


inches long and 1/4 inches thick. If the density of gold is 11. 17 ounces per cubic inch and


the price of gold is $1,303. 80 per ounce, how much will each tile cost? Round your


answer is the nearest dollar.

Answers

To calculate the cost of each tile, we need to first determine the volume of each tile. The length and width of the tile are given as 16 inches, and the thickness is given as 1/4 inches, which can be converted to 0.25 inches. Therefore, the volume of each tile is 16 x 16 x 0.25 = 64 cubic inches.

Next, we need to determine the weight of gold in each tile. Since the density of gold is 11.17 ounces per cubic inch, the weight of gold in each tile is 64 x 11.17 = 715.68 ounces.

Finally, we can calculate the cost of each tile by multiplying the weight of gold by the price of gold per ounce. The price of gold is given as $1,303.80 per ounce, so the cost of each tile is 715.68 x $1,303.80 = $933,526.78. Rounded to the nearest dollar, each tile will cost $933,527.

In summary, each square tile made of solid gold and measuring 16 inches long and 1/4 inches thick will cost approximately $933,527. This cost is based on the density of gold, which is 11.17 ounces per cubic inch, and the price of gold, which is $1,303.80 per ounce.

To know more about  each tile, refer here

https://brainly.com/question/29906822#

#SPJ11

Mrs. Hinojosa had 75 feet of ribbon. If each of the 18 students in her
class gets an equal length of ribbon, how long will each piece be?
Write your answer in 3 ways:
a. using only feet

b. using a whole number of feet and a whole number of inches

c. using only inches

Answers

Using division operation with unit conversions, the length of ribbon that each of the 18 students in Mrs. Hinojosa's class gets is as follows:

a) 4.2 feet.

b) 4 feet and 2 inches

c) 50 inches.

What is division operation?

Division and multiplication operations are used in unit conversions.

Unit conversions involve converting measurements from hours to minutes or seconds, centimeters to meters and miles, etc.

The total quantity of ribbon Mrs. Hinojosa had = 75 feet

1 foot = 12 inches

75 feet = 900 inches (75 x 12)

The number of students in the class = 18

The length of ribbon received by each student = 4.167 feet (75 ÷ 18)

The length of ribbon received by each student ≈ 4 feet and 2 inches

The length of ribbon received by each student in inches only = 50 inches (900 ÷ 18)

Learn more about unit conversions, division, and multiplication at https://brainly.com/question/4158962.

#SPJ1

I need help! Solve for X

Answers

Answer: x=18
4x + 8 = 80
Take away 8 from both sides
4x = 72
Divide 72 by four
X=18

If my refrigerator uses 300 watts when the motor is running, and the motor runs for 30 minutes of every hour, then how much energy does it use per day? How many BTU of energy would that be equal to?

Answers

The amount of energy the refrigerator uses per day is 3.6 kilowatt-hour (kWh)

The refrigerator energy usage per day in BTU is 12283.704 BTU

What is the BTU?

The BTU is an acronym for the British Thermal Unit, which is a measure of heat, which is specified in energy units.

The duration the refrigerator fan runs per hour = 30 minutes

The amount of energy the refrigerator uses every hour = 300 watts × 0.5 hour/hour = 150 watts

The amount of energy the refrigerator uses per hour = 150 watt-hour

The amount of energy the refrigerator uses per day = 24 × 150 watt-hour = 3600 watt-hour = 3.6 kilowatt-hour (kWh)

1 kWh = 3412.14 BTU

Therefore;

3.6 kWh = 3.6 × 3412.14 BTU = 12283.704 BTU

The amount of energy in BTU the refrigerator uses per day = 12283.704 BTU

Learn more on the BTU here: https://brainly.com/question/14252954

#SPJ1

A three digit number is such that twice the hundreds digit is more than the tens digit by 2. The unit digit is thrice the hundred digit. When the digits are reversed the number is increased by 594. Find the number.(5 marks)

Answers

Answer:

Step-by-step explanation:

Let the three-digit number be represented as $abc$, where $a$ is the hundreds digit, $b$ is the tens digit, and $c$ is the units digit.

From the problem, we have two equations:

Equation 1: $2a=b+2$

Equation 2: $c=3a$

We can use these equations to solve for $a$, $b$, and $c$.

Starting with Equation 1, we can isolate $b$ to get $b=2a-2$.

Next, we can substitute Equation 2 into Equation 1 to get $2a=3a-6+2$, which simplifies to $a=8$.

Using this value of $a$, we can now find $b$ and $c$. From Equation 2, we have $c=3a=24$. And from Equation 1, we have $b=2a-2=14$.

Thus, the original three-digit number is $abc=824$.

When we reverse the digits to get $cba=428$, we increase the number by 594, so we have $cba=abc+594=824+594=1418$.

Therefore, the answer is $\boxed{824}$.

Mr. Lance designed a class banner shaped like a polygon shown what is the name of the polygon

Answers

Step 1: Answer

The point (2, 8) is the point (x1, y1) identified from the equation y - 8 = 3(x - 2).

Step 2: Explanation

The equation y - 8 = 3(x - 2) is in point-slope form, which is y - y1 = m(x - x1), where (x1, y1) is the point on the line and m is the slope of the line. In this case, the slope of the line is 3, which means that for every increase of 1 in the x-coordinate, the y-coordinate increases by 3.

Comparing the given equation with the point-slope form, we can see that x1 = 2 and y1 = 8. Therefore, the point (2, 8) is the point identified from the equation.

Reflect (-5,-3) over the x axis then translate the result 2 units downwhat are the final cordinates

Answers

We reflect the point (-5, -3) over the x-axis to get (-5, 3), and then translate this point down by 2 units to get the final coordinates of (-5, 1).

To reflect a point over the x-axis, we keep the x-coordinate the same, but negate the y-coordinate. Thus, reflecting the point (-5, -3) over the x-axis gives us the point (-5, 3).

Next, we need to translate the result 2 units down. To do this, we subtract 2 from the y-coordinate of the reflected point. So, subtracting 2 from the y-coordinate of (-5, 3) gives us the final coordinates:

(-5, 3) translated 2 units down is (-5, 1).

In summary, we can reflect a point over the x-axis by negating the y-coordinate, and translate a point down by subtracting a value from its y-coordinate. So, we reflect the point (-5, -3) over the x-axis to get (-5, 3), and then translate this point down by 2 units to get the final coordinates of (-5, 1).

To know more about reflection, refer to the link below:

https://brainly.com/question/16359605#

#SPJ11

People were asked if they were considering changing what they eat.29% of the people asked said yes.of these, 23% said they were considering becoming vegetarian.what percentage of the people asked said they were considering becoming vegetarian?

Answers

Answer:

66.7%

Step-by-step explanation:

Let people asked bye x.

Then, people considering to change = 29% of x

People considering to become vegetarians = 23% of (29% of x)

                                                                          = 23/100 * 29x/100

                                                                          = 667x/10000

Percentage of people considering to become vegetarians = 667x/10

                                                                                                  = 66.7%

Which set includes ONLY rational numbers that are also integers?

Answers

The set that includes ONLY rational numbers that are also integers is:

{-3, -2, -1, 0, 1, 2, 3, ...}

Which set includes ONLY rational numbers also integers?The set of rational numbers that are also integers is the set of numbers that can be expressed as a ratio of two integers where the denominator is 1. This means that the set includes numbers that are whole numbers, as well as their negatives.

Learn more about integers

brainly.com/question/27908445

#SPJ11

Which number produces a rational number when multiplied by 1?
O A. TI
O в. -3
• C. Т
O D. -1. 41421356

Answers

Number that produces a rational number when multiplied by 1 is option B, -3.

To determine which number produces a rational number when multiplied by 1,

We need to examine each of the given.

Here, options: A. TI , B. -3, C. T, D. -1.41421356

We know,

A rational number can be expressed as a fraction (a/b) where both a and b are integers and b is not equal to 0. When multiplying by 1, the result remains the same. Option B: -3 multiplied by 1 = -3 which is a rational number because it can be expressed as a fraction (-3/1).

Option D: here given number -1.41421356 multiplied by 1 = -1.41421356 and we cannot be easily expressed as a fraction.

Therefore,it's not a rational number.

Thus, the number that produces a rational number when multiplied by 1 is option B, -3.

Learn more about rational number here,

https://brainly.com/question/12088221

#SPJ4

Other Questions
A mosaic of a large cross in a circle. the rest of the scene shows a man with a halo standing in a field with animals and trees. two figures are shown in the sky. this mosaic is an example of ___________ art . a. early byzantine c. late byzantine b. middle byzantine d. jewish please select the best answer from the choices provided a b c d Can someone please help me with these??1. Write three possible topics here from your brainstorming. Do some research on each to determine source availability.2. Choose a general subject for your research paper and write that subject below.3. Limit your subject and write the limited subject below. Trundle wheels are used to measure distances along the ground. The radius of the trundle wheel is 30 cm. Jim wants to work out the distance between two junctions on a road. He rolls the trundle wheel between the two junctions. The trundle wheel rotates exactly 48 times. Work out the distance between the two junctions. Give your answer in metres correct to the nearest metre. Review this reaction:H2SO4+NaOH->?.What are the products? Write three different pairs of coordinate points that form a line segment with a slope greater than 2. Leroy is building a slide for his kids. If the ladder is 5 feet tall and he wants the bottom of the slide to be 12 feet from the ladder, how long does the slide need to be? under a time crunch, you only have time to take a sample of 15 water bottles and measure their contents. the sample had a mean of 20.05 ounces with a sample standard deviation of 0.3 ounces. what would be the 90% confidence interval, when we assumed these measurements are normally distributed? g a buyer offered $150,000 for a home. the seller countered at $160,000. the buyer did not accept the seller's counteroffer. the seller told her broker that she was revoking the counteroffer and accepting the buyer's original offer. which statement is true regarding this situation? suppose you observe the following situation: state of economy probability of state of economy rate of return if state occurs stock a stock b boom .21 .189 .097 normal .74 .158 .076 recession .05 -.246 .042 assume the capital asset pricing model holds and stock a's beta is greater than stock b's beta by .84. what is the expected market risk premium? The cafeteria staff made sandwiches. Each sandwich had either rye or white bread, either ham or turkey, and either cheese or no cheese. The staff made an equal number of each type of sandwich. The sandwiches were placed on a tray. Without looking, Mary will choose a sandwich. What are the chances that Mary will get a sandwich with cheese?Responsesone eighthone sixthone thirdone half Aprils grandmother bought her a set of Russian dolls from St. Petersburg. The dolls stack inside of each other and are similar to each other. The diameters of the two smallest dolls are 1. 9 cm and 2. 85 cm. The scale factor is the same from one doll to the next. April estimates that the volume of the smallest doll is 7 cm^ 3. Determine the volume of the 4th doll Need help with these!! 1. Choose the word most nearly opposite in meaning to - turbidpretentiousdullclearopaque the long-run average cost curve will be upward sloping when the firm is experiencing: a economies of scale. b diseconomies of scale. c constant returns to scale. d diminishing returns. e efficiencies in production. 16.The image of point (3,-5) under the translation that shifts (x, y)to (x-1, y-3) is The Crunchies Snack Food Company packages potato chips in bags. The net weight of the chips in each bag is designed to be 9. 0 oz, with a tolerance of /- 0. 5 oz. The packaging process results in bags with an average net weight of 8. 80 oz and a standard deviation of 0. 12 oz. The company wants to determine if the process is capable of meeting design specifications. What is the Cp and what does it tell you about the capability of the process Write a letter to Your principal outlining three projects you would like to see done in your school Frets are small metal bars positioned across the neck of a guitar so that the guitar can produce notes of aspecific scale. To find the distance a fret should be placed from the bridge, multiply thestring length by 2 " where nis the number of notes higher than the string 's root note. Determine where to place a fret to produce an A note on a C string (5 notes higher) that is 70 cm long. Roundyour answer to the nearest hundredth. a. 52. 44 cmC. 58. 33 cmb. 93. 44 cmd. 74. 92 cm Within a population of butterflies, the color brown (B) is dominant over the color white (b). And, 40% of all butterflies are white. Given this simple information, which is something that is very likely to be on an exam, calculate the following: The percentage of butterflies in the population that is heterozygous. The frequency of homozygous dominant individuals. A meal program accepts online payments by the use of a credit card. for every payment processed, the person is charged a 2% processing fee. if a person made a payment of $23.50, how much was the fee he or she paid?